LSAT and Law School Admissions Forum

Get expert LSAT preparation and law school admissions advice from PowerScore Test Preparation.

User avatar
 Dave Killoran
PowerScore Staff
  • PowerScore Staff
  • Posts: 5852
  • Joined: Mar 25, 2011
|
#41690
Complete Question Explanation
(The complete setup for this game can be found here: lsat/viewtopic.php?t=9037)

The correct answer choice is (B)

If O and R are scheduled for consecutive days, those days must be day 3 and day 4. Thus T, J, and S must be scheduled for day 1, and M must be scheduled for day 2. Answer choices (C), (D), and (E) can be eliminated since they contain either J, or T, or both. Answer choice (A) can be eliminated since F and L are both nurses and two nurses can never teach together. Thus, it follows that answer choice (B) is proven correct by process of elimination.

Get the most out of your LSAT Prep Plus subscription.

Analyze and track your performance with our Testing and Analytics Package.